Monopolos magnéticos de Dirac y cuantización de carga eléctrica fraccionada de quarks

Al aplicar la regla de cuantificación de Dirac para carga eléctrica y magnética, asumo que uno está considerando cargas eléctricas unitarias como los electrones. ¿Cómo se aplica la regla de cuantificación de Dirac para las cargas eléctricas fraccionarias de los quarks?

Respondo esta pregunta en physics.stackexchange.com/q/268709 .

Respuestas (2)

La regla de cuantización de Dirac proviene de integrar el momento angular del campo electromagnético superpuesto de una carga y un monopolo. Este momento angular resulta ser finito e independiente de la distancia h entre la carga y el monopolo. Entonces, el argumento es que si es posible aislar una sola carga fundamental mi y un solo monopolo fundamental gramo en alguna región del espacio, entonces el momento angular total en esa región tiene que ser un múltiplo de . Aquí "aislamiento" significa que la distancia a cualquier otra partícula es h .

Pero tenga en cuenta que el aislamiento de las partículas es fundamental. Si coloca un monopolo cerca de un átomo de hidrógeno, el momento angular total del campo electromagnético se desvanece, porque la densidad del momento angular es de la forma mi × B , que es bilineal en los campos.

Dado que los quarks están confinados, el argumento nunca se puede aplicar a un quark.

Este es un buen punto. No obstante, el argumento se aplica a los QFT donde la cantidad de sabores es lo suficientemente alta como para que no ocurra el confinamiento.
¿Qué pasa si elevas la temperatura por encima de la temperatura de Hagedorn ? Entonces los quarks se desconfinarían y podrías transportar un solo quark alrededor de la cuerda de Dirac.
Su argumento no se aplica en el caso de que mueva un quark alrededor de un monopolo en un círculo más pequeño que el radio de confinamiento o por encima del radio de Hagedorn. Consulte physics.stackexchange.com/questions/268709/… para obtener la respuesta completa.

La lógica es la misma: si q es una carga eléctrica y gramo es una carga magnética, uno debe tener gramo q 2 π Z (en unidades teóricas perezosas). Entonces, si hay una carga magnética más grande, entonces necesariamente hay una carga eléctrica más pequeña. No es necesario suponer que se trata de cargas unitarias para presentar este argumento.